3
$\begingroup$

When reading "Chebyshev centers and uniform convexity" by Dan Amir I encountered the following result which is apparently "known and easy to prove". I'm sure it is, but I can't find a proof and am failing to prove it myself.

The result (slightly simplified) is

If $X$ is a uniformly convex space (i.e. if $||x|| = ||y|| = 1$ with $||x - y|| \geq \epsilon$ then there exists $\delta(\epsilon) > 0$ such that $||\frac{x + y}{2}|| \leq 1 - \delta(\epsilon)$) then for any $x, y$ with $||x|| \leq 1$ and $||y|| \leq 1$, and $||x - y|| \geq \epsilon$, $||\frac{x + y}{2}|| \leq 1 - \delta(\epsilon)$.

Part of the problem is that I think this isn't true without making some additional restrictions to reduce the value of $\delta(\epsilon)$. e.g. by considering $||x|| = 1$ and $y = (1 - \epsilon) x$ you can see that this requires that $\delta(\epsilon) \leq \frac{1}{2} \epsilon$. So I think the true result is probably just that you can choose $\delta$ so that this is true.

I'm sure this should be easy and I'm just missing an obvious trick, but oh well.

$\endgroup$
2
  • $\begingroup$ If you remove the definition in parentheses then the resulting statement is false - just take $x=y$. Are you sure this is what Amir is claiming? $\endgroup$
    – Yemon Choi
    Mar 3, 2012 at 19:31
  • $\begingroup$ Sorry, I missed the $||x - y|| \geq \epsilon$ criterion in the claim (without which it makes no sense because $\epsilon$ didn't appear in it at all). Edited to fix now. Basically the claim is that you can relax the requirement in the definition of uniform convexity that the norm is exactly 1 to that it is $\leq 1$ $\endgroup$ Mar 3, 2012 at 20:27

3 Answers 3

6
$\begingroup$

If the second $\delta(\varepsilon)$ is allowed to differ from the first one, then there is a simple implicit argument: Suppose the contrary, then there is a sequence $X_n$ of 2-dimensional normed spaces satisfying the definition with the same function $\delta(\varepsilon)$ and points $x_n,y_n\in X_n$ with $\|x_n\|\le 1$, $\|y_n\le 1\|$, $\|x_n-y_n\|\ge\varepsilon$ but $\|(x_n+y_n)/2\|\ge 1-\delta_n$ where $\delta_n\to 0$. Since the Banach--Mazur compactum is compact, there is a converging subsequence, and the limit space satisfies the definition for the same $\delta(\varepsilon)$ but contains two points $x,y$ with $\|x\|\le 1$, $\|y\|\le 1$, $\|x-y\|\ge\varepsilon$ and $\|(x+y)/2\|\ge 1$, a contradiction.

In fact, you can always choose the same $\delta(\varepsilon)$ in the second case provided that $\dim X\ge 2$. Suppose the contrary, then there are points $x,y\in X$ such that $\|x\|\le 1$, $\|y\|\le 1$, $\|x-y\|\ge\varepsilon$ but $\|(x+y)/2\|=1-\delta_1$ where $\delta_1<\delta=\delta(\varepsilon)$. We may assume that $X$ is 2-dimensional (otherwise restrict to a 2-dimensional subspace containing $x$ and $y$). Fix $\delta_1$ and from all such pairs $x,y$ choose one that minimize $\big|\|x\|-\|y\|\big|$. I claim that this minimizing pair satisfies $\|x\|=\|y\|$.

Suppose the contrary: let $\|x\|>\|y\|$. Denote $z=(x+y)/2$, $v=(x-y)/2$. If $v$ is proportional to $x$, choose any $v'$ with $\|v'\|=\|v\|$ such that $\|z+v\|\ne \|z\|\pm\|v\|$. Then the points $x'=z+v'$ and $y'=z-v'$ show that $x$ and $y$ did not minimize $\big|\|x\|-\|y\|\big|$. If $v$ is not proportional to $x$, choose a vector $w$ parallel to a supporting line to the unit sphere of $\|\cdot\|$ at the point $v/\|v\|$. Note that $w$ cannot be parallel to a supporting line at $x/\|x\|$, so either $\|x+tw\|<\|x\|$ or $\|x-tw\|<\|x\|$ for a sufficiently small $t>0$. Hence the points $x'=x+tw$ and $y'=y-tw$ or $x'=x-tw$ and $y'=y+tw$ provide a counter-example with $\big|\|x'\|-\|y'\|\big|<\big|\|x\|-\|y\|\big|$.

Thus the minimizing pair satisfies $\|x\|=\|y\|$. Multiplying by $\|x\|^{-1}$ we get a counter-example with $\|x\|=\|y\|=1$.

$\endgroup$
1
  • $\begingroup$ Thanks for the proofs. I must confess I don't understand the first one at all, but that's because I hadn't even heard of the Banach Mazur compaction until now. (this isn't really an area that I know that much about, though I'd like to fix that) $\endgroup$ Mar 4, 2012 at 11:06
3
$\begingroup$

Another book reference:

This is proved as Lemma 9.2 in ``Functional Analysis and Infinite-dimensional Geometry" by Fabian, Habala, Hajek, Montesinos Santalucia, Pelant, and Zizler.

$\endgroup$
0
2
$\begingroup$

In addition to S. Ivanov's proof, I give you a reference to the literature. A proof can be found in Classical Banach Spaces II by J. Lindenstrauss & L. Tzafriri in Section 1.e directly after the definition of uniform convexity on p. 60.

$\endgroup$

Your Answer

By clicking “Post Your Answer”, you agree to our terms of service and acknowledge you have read our privacy policy.

Not the answer you're looking for? Browse other questions tagged or ask your own question.